Number Theory 20

(Junior Balkan MO 2000)
[Bài toán] Tìm n\in \mathbb{Z^+} sao cho n^2+3^n là số chính phương.
                                                           Lời Giải
Đặt n^2+3^n=a^2(1)(a\in \mathbb{Z^+})
\Leftrightarrow (a-n)(a+n)=3^n \Rightarrow\begin{cases}a-n=3^m\\ a+n=3^{n-m}\end{cases} (m\in \mathbb{Z^+})
Ta có: m<c-m\Leftrightarrow 2m<c\Leftrightarrow 2m+1\leq c
\blacktriangleright Xét n=2m+1 \Rightarrow \begin{cases}a-n=3^m\\a+n=3^{m+1}\end{cases}
\Rightarrow a+n=3(a-n)
\Leftrightarrow a=2n\Rightarrow n^2+3^n=4n^2
\Leftrightarrow 3^n=3n^2\Leftrightarrow 3^{n-1}=n^2 \Rightarrow n lẻ.
Đặt n=2k+1(k\in \mathbb{N}) \Rightarrow (3^k)^2=(2k+1)^2\Leftrightarrow 3^k=2k+1
Áp dụng BĐT Bernulli: 1+2k\leq (1+2)^k=3^k
\Rightarrow k\in \{0;1\}\Rightarrow n\in \{1;3\} (Thử lại thỏa mãn)
\blacktriangleright Xét n>2m+1\Rightarrow n\geq 2m+2
Ta có: 2n=3^{n-m}-3^m=9.3^{n-m-2}-3^m3^m\leq 3^{n-m-2}\Leftrightarrow n\geq 2m+2(\text{True})
Áp dụng bất đẳng thức Bernulli:
\Rightarrow 2n\geq 8.3^{n-m-2}=8.(1+2)^{n-m-2}\geq 8[1+2(n-m-2)]=16n-16m-24
\Leftrightarrow 7n\leq 8m+127n\geq 14m+14\Rightarrow 6m+2\leq 0 (vô lí)
Vậy \boxed{n\in \{1;3\}} thỏa mãn \blacksquare

Number Theory 8

(Junior Balkan MO 1998)
Tìm nghiệm nguyên dương của phương trình: x^y=y^{x-y}
                                                                Lời Giải
Ta có: x^y=y^{x-y}\Leftrightarrow x=y^{\frac{x-y}{y}}\in \mathbb{Z}
\Rightarrow x-y\vdots y\Rightarrow x\vdots y
Đặt x=ty(t\in \mathbb{Z^+})
Ta có: ty=y^{\frac{ty-y}{y}}\Leftrightarrow t=y^{t-2} (1)
(*) Nếu y=1\Rightarrow x=1.
(*) Nếu y\geq 2
+, Xét t=1\rightarrow (1)\Leftrightarrow 1=y^{-1}\Leftrightarrow y=1\Rightarrow x=1 (không thỏa mãn)
+, Xét t=2\rightarrow (1)\Leftrightarrow 2=y^0 (vô lí)
+, Xét t=3\rightarrow (1)\Leftrightarrow y=3\Rightarrow x=9
+, Xét t=4\rightarrow (1)\Leftrightarrow y^2=4\Leftrightarrow y=2\Rightarrow x=8
+, Xét t\geq 5. Ta dùng quy nạp, cần phải chứng minh: y^{t-2}>t (2)
• Xét t=5 thì ta có Q.E.D
• Xét t>5. Giả sử y^{t-2}>t, ta phải CM với t+1 thì điều phải chứng minh luôn đúng.
Thật vậy, ta có: y^{t-1}\geq 2^{t-1}=2.2^{t-2}> 2t>t+1
Từ (2)\Rightarrow Trường hợp này không xảy ra.
Vậy nghiệm nguyên dương của phương trình là \boxed {(x,y)=(9;3);(1;1);(8;2)} \blacksquare